Видеоурок: интегральная теорема Муавра-Лапласа

Данная теорема является дальнейшим развитием схемы Бернулли и позволяет работать с диапазонами: какова вероятность, что число успехов будет лежать в пределах указанного отрезка.

Интегральная теорема Муавра-Лапласа. Пусть число испытаний $n$ достаточно велико, а вероятность успеха $0 \lt \,p\, \lt 1$. Пусть также$q=1-p$ — вероятность неудачного испытания. Тогда вероятность того, что число успехов будет лежать в пределах от ${{k}_{1}}$ до ${{k}_{2}}$, можно примерно посчитать по формуле:

\[{{P}_{n}}\left( {{k}_{1}};{{k}_{2}} \right)\approx \Phi \left( \frac{{{k}_{2}}-np}{\sqrt{npq}} \right)-\Phi \left( \frac{{{k}_{1}}-np}{\sqrt{npq}} \right)\]

где

\[\Phi \left( x \right)=\frac{1}{\sqrt{2\text{ }\!\!\pi\!\!\text{ }}}\int\limits_{0}^{x}{{{e}^{-\frac{{{t}^{2}}}{2}}}dt}\]

Функция $\Phi \left( x \right)$ называется функцией Лапласа и содержит в себе интеграл, который не считается напрямую. Как следствие, значения этой функции сведены в таблицу, которую можно загрузить прямо на этой странице.

Разумеется, в таблице приведены не все возможные значения. Для больших значений $x$ (скажем, для $x \gt 6$ ) считают, что $\Phi \left( x \right)\approx 0,5$. Кроме того, функция Лапласа является нечётной, поэтому из неё можно выносить знак «минус»:

\[\Phi \left( -x \right)=-\Phi \left( x \right)\]

Это прямо следует из определения, в котором присутствует определённый интеграл.

Что такое "интегральная теорема Муавра-Лапласса"?

Сегодня мы разберем интегральную теорему Муавра-Лапласа. Это «старшая сестра» второй версии теоремы Муавра-Лапласа, разобранной в прошлом уроке. Во-первых, разберемся, зачем вообще нужна еще одна теорема — интегральная.

Допустим, у нас есть 1000 изделий, о которых известно, что там в среднем есть 10% брака. Однако это не означает, что в партии из 1000 изделий будет ровно 100 бракованных изделий, скорее всего, их будет 101-102 или 98, но не 100. Вероятность того, что будет ровно 100, легко считается при помощи более легкой теоремы Муавра-Лапласа, и вы можете сами убедиться, что она будет велика. В этом случае возникает вопрос: «Какова тогда вероятность, что деталей будет от 95 до 105, либо от 50 до 150?». Считать такую конструкцию при помощи первой версии теоремы Муавра-Лапласса крайне сложно, потому что нам придется отдельно посчитать, какова возможность того, что бракованных изделий будет 50, 51, 51 и так до 150, т.е. сто отдельных однотипных вычислений. Это очень трудоемко и бессмысленно. Вот именно в таких примерах нам на помощь приходит интегральная теорема Муавра-Лапласа. С назначением самой интегральной теоремы все ясно, теперь давайте разберемся с ее формулой.

Вероятность того, что при $n$-испытаниях количество успешных испытаний будет в пределах от ${{K}_{1}}$ до ${{K}_{2}}$ выражается следующей формулой:

\[{{P}_{n}}\left( {{K}_{1}};{{K}_{2}} \right)\approx F\left( \frac{{{K}_{2}}-np}{\sqrt{npq}} \right)-F\left( \frac{{{K}_{1}}-np}{\sqrt{npq}} \right)\]

Сама же функция $F$ называется функцией Муавра-Лапласа, и выглядит она следующим образом:

\[F\left( x \right)=\frac{2}{\sqrt{2\text{ }\!\!\pi\!\!\text{ }}}{{\int\limits_{0}^{x}{e}}^{-\frac{{{t}^{2}}}{2}}}dt\]

Сразу же скажу, что данный интеграл «красиво» не считается, поэтому вместо красивого интегрирования у вас всегда будет в распоряжении таблица значений функции Лапласа, и с помощью этой таблицы, а также некоторых способов, которые мы разберем чуть позже в этом уроке, мы и будем решать все примеры на данную интегральную теорему.

Разумеется, возникает вопрос «А что это за буквы такие — $n$, $q$, $p$?».

С $n$, я думаю, все понятно — это число испытаний.

$p$ — это вероятность успеха в каждом конкретном испытании.

$q$ — по аналогии с формулой Бернули это вероятность провала, т.е. неуспеха в каждом конкретносм испытании. Считатеся она по очень простой формуле:

\[q=1-p\]

Надеюсь, с буквами теперь понятно, поэтому перейдем к решению конкретных примеров.

Задача № 1

Начнем мы с довольно простой задачи, однако уже на ее примеры мы познакомимся с особенностями применения интегральной теоремы Муавра-Лапласса.

Известно, что в среднем 5% студентов носят очки. Какова вероятность того, что из 200 студентов, находящихся в аудитории, окажется не менее 10%, носящих очки?

В первую очередь, давайте запишем саму интегральную теорему Муавра-Лапласса:

\[{{P}_{n}}\left( {{K}_{1}};{{K}_{2}} \right)\approx F\left( \frac{{{K}_{2}}-np}{\sqrt{npq}} \right)-F\left( \frac{{{K}_{1}}-np}{\sqrt{npq}} \right)\]

При этом полезно помнить еще одну формулу:

\[F\left( x \right)=\frac{2}{\sqrt{2\text{ }\!\!\pi\!\!\text{ }}}{{\int\limits_{0}^{x}{e}}^{-\frac{{{t}^{2}}}{2}}}dt\]

Собственно, из-за этого интеграла, присутствуещего в функции Муавра-Лапласса, сама теорема и называется интегральной.

При первом взгляде на эту интегральную теорему многие ученики приходят в шок — уж больно много здесь разных конструкций, корней, вычислений и т.д. На самом деле, все очень просто, и сейчас вы сами в этом убедитесь.

Для начала давайте выпишем все значения. Итак, нам известно следующее:

Далее мы можем найти $\sqrt{npq}$:

\[\sqrt{npq}=\sqrt{200\cdot 0,05\cdot 0,95}=\sqrt{9,5}\approx 3,08\]

Разумеется, такие вычисления выполняются на калькуляторе.

Кроме того, в нашей формуле, в интегральной теореме Муавра-Лапласса, мы наблюдаем выражение $np$ — произведение количества испытаний на вероятность успеха:

\[np=200\cdot 0,05=10\]

Давайте перепишем формулу с учетом всех фактов:

\[{{P}_{n}}\left( {{K}_{1}};{{K}_{2}} \right)\approx F\left( \frac{200-10}{3,08} \right)-F\left( \frac{20-10}{3,08} \right)=\]

\[=F\left( 61,7 \right)-F\left( 3,25 \right)\]

И вот здесь нас поджидает первая проблема: если мы посмотрим на таблицу значений, то значение $3,25$ здесь еще присутствует, но вот числа от $60$ и более здесь вообще не представлены. Для решения этого вопроса предлагаю взглянуть на исходную формулу Муавра-Лапласса:

\[F\left( x \right)=\frac{2}{\sqrt{2\text{ }\!\!\pi\!\!\text{ }}}{{\int\limits_{0}^{x}{e}}^{-\frac{{{t}^{2}}}{2}}}dt\]

При больших «иксах» ${{e}^{-\frac{{{t}^{2}}}{2}}}$ будет очень маленьким числом, т.е. возрастание $x$ дает очень маленькую, стремящуюся к «нулю» добавку к вероятности. Поэтому для всех «иксов», начиная от шести и более примерно считается, что значение функции Лапласса равно $0,5$. $$ $$

Итак, продолжим наше решение:

\[{{P}_{n}}\left( {{K}_{1}};{{K}_{2}} \right)\approx 0,5-0,49942=0,00058=5,8\cdot {{10}^{-4}}\]

Нюансы решения

Как видите, ничего сверхъестественного. Все применение интегральной теоремы Муавра-Лапласса сводится к следующему:

  1. Аккуратно выписать все значения: число испытаний, вероятность и «единицу» «минус» вероятность.
  2. Посчитать корни и величины.
  3. Пробежаться глазами по таблице и найти значение функции в тех точках, которые мы получили.

Однако, как вы понимаете, это была самая простая задача — существуют гораздо более сложные и навороченные. И один из самых «противных» типов заданий на применение интегральной теоремы Муавра-Лапласса состоит в том, что общая вероятность, которую мы обычно рассчитываем по формуле, нам известна, а необходимо найти либо ${{K}_{1}}$, либо ${{K}_{2}}$. Вот именно сейчас такую задачу мы и решим.

Самое обидное, что именно такие чаще всего и попадаются на всяких контрольных, зачетах и экзаменах. Они будут вам встречаться на исследованиях, где необходимо определить какую-нибудь статистическую величину. Поэтому именно сейчас мы попытаемся решить такую задачу.

Задача № 2

Театр, вмещающий 1000 человек, имеет два разных входа, каждым из которых любой зритель может воспользоваться с равной вероятностью. Около каждого входа имеется свой гардероб. Сколько мест должно быть в каждом гардеробе, чтобы с вероятностью в 0,99 любой зритель смог раздеться в том гардеробе, в который он обратился сразу после входа в театр.

Я думаю, очевидно, что в данной задаче общее количество испытаний, т.е. человек, которые придут в театр, не более 1000 — $n=1000$.

Всего входов два, при этом в каждый с одинаковой вероятностью входит один и тот же человек — $p=\frac{1}{2}$.

Следовательно, $q=1-\frac{1}{2}=\frac{1}{2}$.

Кроме того, общая возможность того, что при 1000 испытаний количество успеха попадет в искомый нами диапазон, равно 0,99 — ${{P}_{1000}}\left( {{K}_{1}};{{K}_{2}} \right)=0,99$. Остается разобраться с числами ${{K}_{1}}$ и ${{K}_{2}}$, т.е. границами диапазона. ${{K}_{1}}$ — наименьшее количество людей, которые могут обратиться в данный гардероб. Очевидно, будет «ноль», потому что меньше нуля прийти не может — ${{K}_{1}}=0$. Остается вопрос: «Чему равно ${{K}_{2}}$?». Именно это нам и нужно найти по условию.

Опять запишем интегральную теорему Муавра-Лапласса:

\[{{P}_{1000}}\left( {{K}_{1}};{{K}_{2}} \right)\approx F\left( \frac{{{K}_{2}}-np}{\sqrt{npq}} \right)-F\left( \frac{{{K}_{1}}-np}{\sqrt{npq}} \right)\]

Посмотрим:

\[np=1000\cdot \frac{1}{2}=500\]

\[\sqrt{npq}=\sqrt{1000\cdot \frac{1}{2}\cdot \frac{1}{2}}=\sqrt{250}=5\sqrt{10}=15,8\]

Подставим все полученные числа в формулу, с учетом того, что ${{K}_{1}}=0$:

\[{{P}_{1000}}\left( {{K}_{1}};{{K}_{2}} \right)\approx F\left( \frac{{{K}_{2}}-500}{15,8} \right)-F\left( \frac{0-500}{15,8} \right)=0,99\]

Теперь внимательно посмотрим на эту формулу. Отдельно посчитаем значение функции Муавра-Лапласса в следующей точке:

\[F\left( \frac{-500}{15,8} \right)=-F\left( 31,6 \right)=0,5\]

Итого переписывая, мы получаем:

\[F\left( \frac{{{K}_{2}}-500}{15,8} \right)+0,5=0,99\]

\[F\left( \frac{{{K}_{2}}-500}{15,8} \right)=+0,49\]

Единственный способ, при помощи которого можно решить этот пример — это взять таблицу значений функции Муавра-Лапласса и посмотреть, когда она равна $0,49$, при каком $x$. Проблема состоит в том, что точного значения мы не найдем. Однако есть значение функции Муавра-Лапласса в точках $2,32$ и $2,34$ :

\[F\left( 2,32 \right)=0,48983\]

\[F\left( 2,34 \right)=0,49036\]

Где-то между ними лежит наша искомая величина $0,49$. А между числами $2,32$ и $2,34$ лежит величина $2,33$. Так и запишем:

\[\frac{{{K}_{2}}-500}{15,8}=2,33\]

Теперь нам осталось решить простейшее уравнение:

\[{{K}_{2}}-500=2,33\cdot 15,8\]

\[{{K}_{2}}-500=36,8\]

\[{{K}_{2}}\approx 536,8=537\]

Ответ: 537.

Каверзные вопросы

Подождите, есть несколько вопросов. Во-первых, почему мы так легко вынесли «минус» из функции Лапласса наружу, а во-вторых, почему мы постоянно пользуемся калькулятором?

Давайте для начала посмотрим на формулу функции Муавра-Лапласса:

\[F\left( x \right)=\frac{2}{\sqrt{2\text{ }\!\!\pi\!\!\text{ }}}{{\int\limits_{0}^{x}{e}}^{-\frac{{{t}^{2}}}{2}}}dt\]

Это, прежде всего, интеграл от «нуля» до $x$ в прелах четной функции, поэтому если перед $x$ внезапно появится «минус», мы можем поменять местами верхние и нижнее пределы интегрирования, при том перед самим интегралом также появится знак «минус», и больше никаких изменений не будет. Это одно из ключевых свойств определенного интеграла.

Кроме того, в таблице значений все аргументы функций приведены именно в виде десятичных дробей, поэтому считая значение функции, мы просто обязаны перевести то, что стоит у нас внутри скобок, в десятичную дробь, в том числе с помощью калькулятора.

В заключение посмотрим еще одну задачку, в которой мы не только еще раз отработаем использование стандартной формулы, но и вспомним, что такое вторая версия теоремы Муавра-Лапласса, отличная от интегральной, и в каких ситуациях она применяется.

Задача № 3

Радиотелеграфная станция передает цифровой текст. В силу наличия помех каждая цифра независимо от других может быть неправильно принята с вероятностью 0,01. Найдите вероятность того, что в принятом тексте, содержащем 1100 цифр, будет:

а) 15 ошибок;

б) менее 20 ошибок.

Решение пункта б)

Что касается б), то тут все вполне очевидно — это чистейшая теорема Муавра-Лапласса, причем интегральная. Так и запишем:

\[n=1100\]

\[p=0,01\]

\[q=0,99\]

\[{{K}_{1}}=0\]

\[{{K}_{2}}=19\]

Теперь запишем интегральную формулу Муавра-Лапласса:

\[{{P}_{n}}\left( {{K}_{1}};{{K}_{2}} \right)\approx F\left( \frac{{{K}_{2}}-np}{\sqrt{npq}} \right)-F\left( \frac{{{K}_{1}}-np}{\sqrt{npq}} \right)\]

Посчитаем:

\[np=1100\cdot 0,01=11\]

\[\sqrt{npq}=\sqrt{11\cdot 0,99}=\sqrt{\frac{11\cdot 11\cdot 9}{100}}=\sqrt{\frac{{{11}^{2}}\cdot {{3}^{2}}}{{{10}^{2}}}}=\frac{11\cdot 3}{10}=3,3\]

Осталось подставить числа в формулу:

\[{{P}_{1100}}\left( 0;19 \right)\approx F\left( \frac{19-11}{3,3} \right)-F\left( \frac{0-11}{3,3} \right)=\]

\[=F\left( 2,42 \right)+F\left( 3,33 \right)=\]

\[=0,49224+0,49960=0,99184\approx 0,99\]

Ответ: 0,99

Решение пункта а)

А теперь давайте разберемся с пунктом а). В нем от нас требуется, чтобы при тех же исходных данных, вычислить, что в итоге появится ровно 15 ошибок.

Очевидно, что это идеальная задача для применения второй версии теоремы Муавра-Лапласса — не интегральной. Давайте я ее запишу:\[\]

\[{{P}_{n}}\left( K \right)\approx \frac{1}{\sqrt{npq}}\cdot \varphi \left( \frac{K-np}{\sqrt{npq}} \right)\]

Выпишем известные данные:

\[n=1100\]

\[p=0,01\]

\[q=0,99\]

Решим:

\[{{P}_{1100}}\left( 15 \right)\approx \frac{1}{3,3}\cdot \varphi \left( \frac{15-11}{3,3} \right)\approx 0,303\cdot \varphi \left( 1,212 \right)\approx \]

\[\approx 0,303\cdot 0,1919\approx 0,058\]

Ответ: 0,058.

Ключевые моменты

Вот и все, что я хотел вам рассказать об интегральной теореме Муавра-Лапласса, такой, на первый взгляд сложной, но очень простой на практике. Все, что вам необходимо — это

  1. Знать сами формулы для обеих теорем Муавра-Лапласса, в том числе, интегральной.
  2. Грамотно считать корни и элементы $np$, которые являются матожиданием.

В ближайшее время я размещу на своем сайте целый комплект задач, посвященный теоремам Муавра-Лапласса, в том числе, интегральной. Поэтому присоединяйтесь к нам на YouTube, ставьте лайки и пишите комментарии. До новых встреч!

Смотрите также:
  1. Локальная теорема Муавра — Лапласа
  2. Схема Бернулли. Примеры решения задач
  3. Основное тригонометрическое тождество
  4. Профильный ЕГЭ-2022, задание 6. Геометрический смысл производной
  5. Иррациональные неравенства. Часть 2
  6. Задача B4: обмен валют в трех различных банках